Juris Education logo

Sign up to our Newsletter

Bar exam practice questions & answers.

new york state bar exam practice questions

Reviewed by:

David Merson

Former Head of Pre-Law Office, Northeastern University, & Admissions Officer, Brown University

Reviewed: 2/16/24

As your bar exam looms near, it's perfectly normal to feel overwhelmed by the multitude of statutes, cases, and complex legal terms you need to grasp. Sure, the multitude of legal concepts might feel like a handful, but we’re here to help! This guide will delve into some bar exam practice questions to provide a clearer perspective on the exam.

We're here to help you study! We've gathered a bunch of practice questions for the bar exam. And don't worry, we've also included all the answers and some explanations. Ready to start?

The bar exam, a critical milestone in every budding lawyer's journey , is notorious for its difficulty. But remember, with a strategic approach, consistent practice of real MBE practice questions, the perfect bar exam practice test, and the right mindset , you can significantly improve your chances of success.

Practice Questions and Answers

Bar exam prep can be expensive, so to start you off on the right foot, we’ll explore some free bar exam sample questions and answers, with a special emphasis on multiple-choice questions:

1. Torts Law: Understanding Liability and Damages

Question : Tom, in a moment of inattention, collides with Jerry's car, inflicting $2000 worth of damage. Jerry decides to sue Tom for negligence. Tom admits his mistake but refuses to pay, asserting that Jerry's car was already in a state of disrepair. Can Jerry still claim damages?

  • Yes, Jerry can claim the full amount of $2000.
  • No, Jerry cannot claim any damages because his car was already in disrepair.
  • Yes, but Jerry can only claim half the amount ($1000) due to pre-existing damages.
  • No, Jerry can only claim damages if Tom had intentionally caused the accident.

Correct Answer : A) Yes, Jerry can claim the full amount of $2,000.

Explanation

Tort cases are commonly referenced in MBE bar exam questions. In a tort case, such as this, Jerry can recover damages as long as he can prove the cost of repairs or the diminished value of his car due to the accident. Tom's belief about pre-existing damage doesn't negate his liability for the damages he caused. 

While the state of the car before the accident might be relevant in assessing the degree of damage caused by the accident, it does not absolve Tom of his responsibility to pay for the harm he directly caused.

2. Criminal Law: Grasping the Concept of Intent

Question : Can Ann be convicted of burglary if her defense is that she entered the building solely to seek refuge from a sudden storm?

  • Yes, Ann can be convicted of burglary even if she entered the building to seek refuge from a sudden storm.
  • No, Ann cannot be convicted of burglary if she entered the building solely to seek refuge from a sudden storm.
  • Maybe, it depends on whether Ann had a history of committing crimes.
  • It depends on the severity of the storm that Ann encountered.

Correct Answer : B. No, Ann cannot be convicted of burglary if she entered the building solely to seek refuge from a sudden storm.

man handcuffed behind back

Ann cannot be convicted of burglary if she can prove that she entered the building solely to seek refuge from a sudden storm. 

Burglary requires the intent to commit a crime upon entering a building, and seeking shelter from a storm is not a criminal act . Therefore, if Ann can validate her claim of seeking shelter, she lacks the requisite intent for a burglary conviction.

3. Constitutional Law: Freedom of Speech

Question : Which of the following statements is true regarding the state law that prohibits residents from criticizing the governor on social media?

  • Yes, the law violates the First Amendment rights of the residents.
  • No, the law does not violate the First Amendment rights of the residents.
  • Maybe, it depends on the type of social media platform being used.
  • It depends on the popularity of the governor.

Correct Answer : A. Yes, the law violates the First Amendment rights of the residents.

This is because the First Amendment of The Constitution protects the right to freely criticize the government, which would include government officials such as a state governor. A law that prohibits such criticism would likely be seen as infringing on these protected rights. 

4. Contracts Law: The Essentials of a Contract

Question : Which of the following statements is true regarding the scenario where Jane proposed to sell her car to John for $5,000, but sold the car to someone else the very next day without receiving any acceptance or consideration from John?

  • Yes, John can sue Jane for breach of contract because she made an offer to him.
  • No, John cannot sue Jane for breach of contract because no contract was formed between them.
  • Maybe, John can sue Jane for breach of contract because he needed time to think it over.
  • It depends on the type of contract Jane proposed.

Correct Answer : B. No, John cannot sue Jane for breach of contract because no contract was formed between them.

two women signing a

Many of the bar test questions you’ll see will involve contract law since it’s such an important part of the legal field. For a contract to be formed, it requires an offer, acceptance, and consideration. In this scenario, Jane made an offer to sell her car to John, but John did not accept the offer or provide any consideration to Jane. 

Therefore, there was no legally binding contract formed between them, and Jane was not obligated to sell the car to John. Since there was no contract, there can be no breach of contract, and John cannot sue Jane. 

Option A is incorrect because an offer alone does not constitute a contract. Option C is incorrect because John's need for more time to think it over is not relevant to the formation of a contract. Option D is incorrect because the type of contract proposed by Jane is irrelevant when no contract was formed.

5. Property Law: The Obligations of a Finder

Question : Is Jim legally allowed to keep the $1000 he found in a wallet in a public park, even though the owner later posted flyers in search of the wallet?

  • Yes, Jim is legally allowed to keep the money as he found it in a public place.
  • No, Jim is not legally allowed to keep the money as it belongs to the owner of the wallet.
  • Maybe, it depends on whether Jim reported the found wallet to the authorities.
  • It depends on whether Jim made any attempts to locate the owner of the wallet.

Correct Answer : B. No, Jim is not legally allowed to keep the money as it belongs to the owner of the wallet.

Jim is not legally allowed to keep the $1000 he found in the wallet in a public park. Property law requires finders of lost property to make a reasonable effort to return it to its rightful owner. Since the owner is known and has posted flyers in search of the wallet, Jim is legally bound to return the wallet and the money to the owner. 

Keeping the money without attempting to locate the owner is considered theft. Therefore, Jim is not allowed to keep the money, and he must make a reasonable effort to return the wallet to its rightful owner.

Hopefully going over these MBE practice questions in depth has helped you understand them better and offered insights into how to best approach them! Continue going over sample MBE questions and past bar exams to get familiar with various question types and feel prepared for test day! 

FAQs: Bar Exam Questions

Here are some frequently asked questions related to bar exam questions.

1. What Kind of Questions Are on the Bar Exam?

A standard bar exam comprises a combination of multiple-choice questions, often referred to as the Multistate Bar Examination (MBE), essay questions, and performance tests. These questions encompass a broad array of legal areas, including, but not limited to, constitutional law, criminal law, civil procedure, contracts, and property law.

2. Is the Bar Exam Actually Hard?

Yes, the bar exam can feel very hard because it covers a lot of topics, there's limited time, and it's really important. But, with good studying, lots of practice , and a never-give-up attitude , many people do succeed.

3. How Can I Practice for the Bar Exam?

Preparation for the bar exam calls for a well-rounded approach. This includes gaining a firm understanding of legal principles, committing key facts and cases to memory, and rigorously working through bar exam practice questions. 

Using a mix of bar exam practice multiple choice questions and essays, as well as reviewing model answers, can substantially enhance your legal analytical abilities, as well as improving your speed and accuracy in test-taking. 

Additionally, consider joining a study group or enlisting the help of a tutor for structured guidance and support.

4. What Essay Questions Are on the Bar Exam?

Essay questions on the bar exam typically present a hypothetical scenario and require you to apply legal principles to analyze it. These questions are designed to test your ability to identify relevant legal issues, apply the law accurately, and articulate a logical and well-structured argument. 

Some jurisdictions also include Multistate Essay Examination (MEE) questions, which test knowledge across a wide range of legal topics.

Final Thoughts

Facing the bar exam may feel like a herculean task, but take heart in knowing that countless others have successfully surmounted this obstacle, and you certainly can too. 

Regular, methodical practice with bar exam questions and answers will not only deepen your understanding of the law but also refine your analytical skills and boost your self-confidence.

As you forge ahead with your preparation, keep this essential principle in mind - practice does not merely make perfect, perfect practice makes perfect. View these bar exam sample questions as stepping stones that enable you to refine your techniques, rectify errors, and devise a strategy that best suits your individual learning style.

Ultimately, your success in the bar exam will stand as a testament to your unwavering dedication, determination, and mastery of legal knowledge. So, keep practicing, stay focused, and before long, you'll be on your way to joining the esteemed fraternity of licensed attorneys. 

Your journey may be tough, but the destination will certainly be worth it. Good luck!

new york state bar exam practice questions

Schedule A Free Consultation

You may also like.

How to Become a Litigator

How to Become a Litigator

How to Become an Intellectual Property Paralegal

How to Become an Intellectual Property Paralegal

image of youtube logo

New York Bar Exam

Written by: Nik Ventouris

Man holding exam paper and pencil in New York.

Last updated: April 5, 2024

Passing the  New York bar exam  is an essential part of being admitted into the state’s bar and beginning a successful career practicing law.

In this guide, we’ll break down everything you need to get started, including:

  • The bar exam’s eligibility criteria, content, and structure
  • The steps you can take in order to give yourself the best chance of passing on your first attempt
  • The additional requirements you’ll need to satisfy in order to become admitted into the state’s bar

We’ll also briefly look at the process of getting admitted without a completed law degree.

Recommended:  Interested in getting started? We recommend Kaplan’s bar exam preparation materials, which come with a free retake guarantee, interactive and flexible learning, and lawyer-led tutoring.

4.7 out of 5 stars

New York Bar Exam Overview

The bar examination in the state of New York is administered via the New York State Board of Law Examiners, which has been in existence since 1894 – over 125 years ago.

The State Board’s operations are overseen by a five-member Board of Attorneys, which is appointed by the New York Court of Appeals.

If you are interested in sitting the bar exam in NY, you should know that exams are administered twice per annum, on the last consecutive Tuesday and Wednesday of each February and July.

Registrations need to be carried out between October 1 and October 31 for the February exam cycle and between March 1 and March 31 for the July exam cycle.

Keep in mind that in order to be eligible for the NY bar exam, you will need to satisfy one of the following criteria according to Section 520 of the Rules of the Court of Appeals:

  • ABA Approved Law School Study (JD Graduates) : Attend and graduate from a law school in the US, which at all times during your attendance is approved by the American Bar Association (ABA).
  • Law Office Study:  Complete a combination of ABA-approved law school study and law office study.
  • Unapproved Law School Study:  Graduate from an unapproved law school in the US with a JD degree and practice in a jurisdiction where you are admitted for 5 out of the seven years preceding your NY bar exam application.
  • Foreign Law School Study:  Complete a program of study at a law school outside the US that is durationally and substantially commensurate to an ABA-approved law school.
  • Pro Bono Scholars Program : If you are currently in your last year of JD at an ABA-approved law school, you may be able to qualify to sit for the February bar exam in return for devoting your last semester of study to performing pro bono legal services through an approved program.

Note: First-time applicants who have graduated with a JD degree from a New York State law school will be given first priority to select their desired location when registering for the bar exam.

New York Exam Structure

Since 2016, the state of New York has adopted the Uniform Bar Exam (UBE), which adheres to the following structure:

  • Multistate Performance Test (MPT): Two 90-minute Multistate Performance Test Questions (20%)
  • Multistate Essay Examination (MEE): Six 30-minute Multistate Essay Exam questions (30%)

Day 2: 

  • Multistate Bar Examination (MBE): 200-question multiple-choice exam (50%)

The UBE exam is scored out of 400 points, 266 of which need to be answered correctly in order for a candidate to receive a passing mark.

Moreover, candidates will be required to pass the following supplementary exams and complete the following course in order to be eligible for the New York bar:

Multistate Professional Responsibility Examination (MPRE)

  • New York Law Course (NYLC)
  • New York Law Exam (NYLE)

Below, we’ve broken down the content and structure of each exam-related requirement in more detail.

Multistate Performance Test (MPT)

The MPT is made up of two 90-minute parts, and the materials for each part contain a “File” and a “Library.”

The File consists of source documents that contain all of the facts of a specific case.

As the examinee, the assignment that you will need to complete will be described in a memorandum from a supervising attorney.

The File can also include things like transcripts of interviews, depositions, pleadings, trials, client documents, newspaper articles, police reports, and any other similar documents.

Keep in mind that irrelevant information is generally included, and facts are sometimes incomplete, ambiguous, or even conflicting.

This is meant to mirror legal practice, in which a client’s or supervising attorney’s version of events may be unreliable or entirely incomplete. In such a scenario, you will be expected to recognize when facts are inconsistent or missing in the exam and identify sources of additional facts.

The Library part contains things like cases, statutes, and regulations, some of which may also not be relevant to the task that you will be assigned. You will need to be able to extract the legal principles required to analyze the legal problem you’ll be presented with and perform the requested task.

For more information, you can have a look at the National Conference of Bar Examiners’ Instructions for Taking the MPT document.

Free summaries of MPTs from recent examinations are also offered (e.g., MPT Summaries of 2023 ).

Multistate Essay Exam (MEE)

The MEE consists of six 30-minute sections that can cover a variety of legal areas, including:

  • Civil Procedure
  • Business Associations (e.g., Agency and Partnership, LLCs, Corporations, etc.)
  • Conflict of Laws
  • Constitutional Law
  • Contract Law
  • Criminal Law
  • Trust and Estates

Keep in mind that questions can often include issues in more than one area of law. This means that it’ll be important to ensure that you have a “wide” range of understanding rather than a deep focus on one or two subjects.

Note: For more information, have a look at the NCBE’s official Instructions for Taking the MEE document or check out a few official MEE questions .

Multistate Bar Exam (MBE)

The MBE is made up of 200 multiple-choice questions, which are broken down into 175 scored questions and 25 unscored questions.

The exam is broken down into two three-hour sections, with one administered in the morning and one in the afternoon, both of which contain 100 questions each.

Keep in mind that there are no scheduled breaks during either one of the sections.

All 175 scored questions on the exam are distributed evenly, with 25 questions being attributed to each of the following seven topics:

  • Real Property
  • Criminal Law and Procedure

Each question on the MBE exam will have four potential answers. You should choose what you believe is the best answer — keep in mind that scores are based on the number of questions answered correctly, and points are not subtracted for answering incorrectly.

If you want to have a look at the exact format, you can have a look at the NCBE’s official MBE Sample Test Questions document.

Recommended Course

Interested in getting started? We recommend Kaplan’s bar exam preparation materials, which come with a free retake guarantee, interactive and flexible learning, and lawyer-led tutoring.

The MPRE is made up of 60 multiple-choice questions (50 scored questions and 10 unscored questions) and is administered via Pearson VUE.

Like the MBE exam, each question offers four possible answers, one of which is correct.

The exam’s content is based on law that relates to the conduct and discipline of attorneys and judges and includes the American Bar Association’s (ABA) Model Rules of Professional Conduct and Model Code of Judicial Conduct, as well as important constitutional common law and generally accepted principles.

In relation to questions of professional responsibility in the context of evidentiary issues (e.g., litigation sanctions, attorney-client evidentiary privilege, etc.), the Federal Rules of Civil Procedure and the Federal Rules of Evidence will apply  unless otherwise stated .

Note:  You will have two hours to complete the MPRE exam in its entirety.

For more information, we recommend having a look at the NCBE’s Official  Sample Test Questions  document. A holistic  Subject Matter Outline  is also offered.

NYLC and NYLE

Following a statement from the Court of Appeals, the New York State Board of Law Examiners (BOLE) created the New York Law Course (NYLC), as well as the New York Law Exam (NYLE) — both of which relate to NY-specific law.

Below, we’ve broken down everything you need to know for each one.

The NYLC is an online course that covers important aspects of the New York legal system, including:

  • Administrative Law
  • Business Relationships
  • Matrimonial and Family Law
  • Professional Responsibility
  • Trusts, Wills, and Estates

The course entails around 15 hours of videotaped lectures that contain embedded questions. Keep in mind that each lecture’s questions will need to be answered correctly in order to begin the next one.

The NYLE is a 50-question, multiple-choice, open-book exam that is offered four times per annum.

Keep in mind that failing the NYLE will result in you having to retake both the NYLC and the NYLE.

BOLE notes that the NYLE exam is rigorous even though it’s administered in an open-book format. This means that, in addition to the NYLC, it is recommended that you have a look and take some time going over the revised  Course Materials  that are offered.

Note:  The New York State Board of Law Examiners additionally offers a  Sample Questions  document with 20 questions.

How to Pass the NY Bar Exam

In order to give yourself the best chance of passing the New York UBE exam on your first attempt, it’s important to take enough time to make sure that you are adequately prepared.

New York Bar Exam Tips

Despite the infamous difficulty of NY’s bar exam, passing it on your first attempt is definitely possible with the right tools and preparation techniques.

Below, we’ve broken down a few key tips that should aid you in your licensing journey:

  • Start Strong in Your First Year:  This is important due to the fundamental areas of law practice that the first year of law school generally covers, such as Contract Law, Tort, and Constitutional Law
  • Leverage Academic Support : If your law school offers additional support services that are aimed at improving your preparation, we recommend utilizing these as much as possible. This can involve one-on-one tutoring, academic counseling, and mock exam programs
  • Obtain Practical Legal Experience : If possible, we recommend gaining as much practical experience as you can during law school (i.e., through summer internships, mock trials, etc.). This is because this can go a long way in cementing your legal knowledge
  • Adopt a Broad Study Approach:  When studying for the bar, you should aim for a wide-ranging understanding of various subjects rather than an in-depth study of a few. This is important due to the huge amount of content that you will need to learn and/or will be tested on

Most importantly, make sure you remain consistent so that you do not have to resort to last-minute cramming.

Research has shown that we are able to retain a higher degree of information – and more easily – when revisiting old content in comparison to when learning it for the first time. This is known as Ebbinghaus’s Forgetting Curve.

Other New York Bar Requirements

Apart from qualifying for and passing the New York bar exam, there are several other requirements you will need to meet in order to become a licensed attorney in the state, including:

  • Completing the Mandatory 50-Hour Pro Bono Requirement
  • Satisfying the Skills Competency Requirement
  • Providing Proof of Moral Character

1. 50-Hour Pro Bono Requirement

In accordance with Rule 520.16 of the Rules of the Court of Appeals, every applicant admitted to the New York State Bar on or after January 1, 2015, shall need to have completed at least 50 hours of “qualifying” pro bono work. This can be done within one of the US’s jurisdictions or in a foreign country.

Qualifying pro bono work is defined in section 520.16 (b), which includes:

Assisting in the provision of legal services without charge:

  • Persons of limited means
  • Nonprofit organizations
  • Individuals, groups, or organizations seeking to secure or promote access to justice (e.g., civil rights, civil liberties, etc.)

Assisting in the provision of legal assistance in public service for a judicial, executive, or legislative government entity (i.e., judge, prosecutor, etc.).

Keep in mind that supervision is required, and as such, all qualifying pro bono work will need to be performed under the supervision of:

  • A member of a law school faculty
  • An attorney admitted to practice and in good standing in a US jurisdiction
  • A judge or attorney employed by the court system (e.g., in the case of a clerkship)

Note:  Work that relates to partisan political activities does not qualify for New York’s 50-hour pro bono requirement.

2. Skills Competency Requirement

In accordance with Section 520.18 of the Rules for the Admission of Attorneys and Counselors at Law, all candidates are required to establish that they have acquired the skills and professional values that are necessary to competently practice law.

This requirement can be satisfied via a variety of different methods, such as by:

  • Submitting a certificate from an ABA-approved law school that confirms that it has incorporated into its curriculum the skills and professional values that are required for graduates’ “basic competence and ethical participation” in legal practice
  • Submitting a certificate from an ABA-approved law school that confirms you enrolled in and successfully completed 15 credit hours of practice-based experiential courses that were designed to foster the development of professional and ethical competencies
  • Completing the Pro Bono Scholars Program of section 520.17, which is a voluntary component of legal education that allows students in their final semester of study to become involved in work that relates to amplifying individuals’ access to justice
  • Completing a qualifying apprenticeship. This is a six-month full-time apprenticeship in a law office in the US under the supervision of one or more attorneys who have been licensed for a minimum of two years. Keep in mind that the apprenticeship does not have to be paid
  • Having practiced in another US jurisdiction or in a foreign jurisdiction outside of the US. Keep in mind that you will need to submit proof that you have been operating for at least one year and are in good standing with your jurisdiction in order to qualify

Since the state’s skill competency requirement can be satisfied via any of the methods mentioned above, most candidates end up picking the route that organically makes sense for them, depending on their existing credentials and short-term plans.

3. Proof of Moral Character

Every applicant for the New York bar will need to provide proof of good moral character.

This is done by submitting affirmations from reputable sources to a special committee appointed by the Appellate Division of the Supreme Court that confirm that you have the moral character and fitness needed to operate ethically as an attorney in accordance with section 90 of the Judiciary Law.

Keep in mind that the Appellate Division decides how many affirmations are needed, as well as who qualifies to provide them (e.g., who can be a “reputable source”).

Note:  Affirmations alone aren’t a guarantee that you will pass this requirement, and the Appellate Division can choose to conduct further investigations if necessary before making a final decision.

Admission on Motion (Reciprocity)

Section 520.10 of the Rules of the Court of Appeals for the Admission of Attorneys and Counselors at Law allows admission “on motion” or reciprocity without examination under specific circumstances.

This means that if you are currently licensed and in good practice in another US jurisdiction or a foreign country, you may be able to become admitted into the NY bar without having to retake an exam (a $400 fee is levied for this).

Let’s take a look at a few of the scenarios in which the Appellate Court can decide to admit applicants in motion/without an exam:

  • Applicants that have been admitted to practice in the highest law court in any other state or territory of the US. This is generally each state’s supreme court
  • Applicants that have been admitted to practice in another country whose judicial system is based upon the principles of English common law (e.g., the UK, Cyprus, Singapore, etc.), are currently admitted to the bar in such jurisdiction and have practiced for at least five of the seven years immediately preceding your application
  • Applicants that have been employed in any state or US territory as a full-time member of the law faculty teaching in an ABA-approved law school and have attained the rank of professor or associate professor for at least five of the seven years preceding the application

Note:  There are certain jurisdictions that have entered into specific formation reciprocity agreements with New York and have established specific standards for admission on motion. These can be more lenient than the general rules for admission discussed above (e.g., New Jersey and New York, etc.).

A full list of reciprocity states for New York can be found below:

For more information, you can have a look at the New York State Board of Law Examiners’  Admission on Motion/Reciprocity  page.

New York Bar Exam FAQ

Can you take the bar without going to law school in new york.

Yes and no. You can be admitted into the New York bar without sitting an examination as a candidate “in motion,” but that would assume that you have practiced and/or studied in another jurisdiction. If this is not the case, you can choose to complete one year of law school in combination with three years of law office study (Section 520.4), but you will not be able to omit law school entirely.

How long is the New York bar exam?

New York administers the UBE exam, which spans over two separate days. Having said that, you will also need to complete additional exam requirements, including the MPRE, the NYLC, and the NYLE. For more New York bar exam information, see our  overview  above.

How hard is it to pass the New York State bar examination?

Passing the New York  bar exam  is undoubtedly very challenging, but it is definitely doable with the right preparation. As of the latest data issued by the New York State Board of Law Examiners, first-time takers from NY ABA law schools had an 81% pass rate. This was 85% for out-of-state ABA law schools and 52% for foreign-educated takers.

How much does the NY bar exam cost?

This will depend on where you studied law. If you studied in the US, your application fee will be $250; this is $750 for applicants who studied outside the US. You will also need to pay $27 for taking the NYLE.

Banner

Bar Exam Preparation Resources

  • About This Guide
  • Access Policies & Hours
  • Boards of Bar Examiners (Bar Admission Information & Requirements)
  • Bar Exam Study Apps
  • Bar Examination Review Materials & Questions

Law School Exam Practice Questions & Answers

  • Weekly Study Plan
  • Academic and Bar Success Initiatives
  • Search Our Catalog This link opens in a new window
  • Library Home Page This link opens in a new window

Contact A Librarian

212.431.2332

[email protected]

Chat (MS Teams)

Library Hours

new york state bar exam practice questions

PLEASE NOTE : These series contain multiple choice questions designed for law school exams . They are not intended for bar exam study, although they may still prove useful.

SERIES:   Questions & Answers: Multiple Choice and Short Answer Questions and Answers

Questions & Answers: Civil Procedure

Questions & Answers: Constitutional Law

Questions & Answers: Contracts

Questions & Answers: Criminal Law

Questions & Answers: Evidence

Questions & Answers: Family Law

Questions & Answers: Torts

SIEGEL'S SERIES: Essay and Multiple-Choice Questions and Answers

Siegel's Civil Procedure: Essay and Multiple-Choice Questions and Answers

Siegel's Constitutional Law: Essay and Multiple-Choice Questions and Answers

Siegel's Contracts: Essay and Multiple-Choice Questions and Answers

Siegel's Property: Essay and Multiple-Choice Questions and Answers

Siegel's Torts: Essay and Multiple-Choice Questions and Answers

  • << Previous: Bar Examination Review Materials & Questions
  • Next: MPRE >>
  • Last Updated: Jan 5, 2024 5:58 PM
  • URL: https://libguides.nyls.edu/barexam

logo for barexam

Pass The New York Bar Exam

Pass the New York bar exam with confidence! Start with our free New York Multistate Bar Exam practice questions below. We’ve selected 15 New York MBE free sample questions among the 7 categories of law tested on the New York Multistate Bar Examination: Civil Procedure, Constitutional Law, Contracts, Criminal Law & Procedure, Evidence, Real Property, Torts. Be sure to review your questions and answers with explanations at the end of the test!

You can then take the New York MBE free trial practice test again or register and gain access to all of our updated New York Multistate Bar Exam Practice Exams in Study Mode and Timed Exam Mode using our proprietary user-friendly interface, plus access over 1,000 flashcards with legal terms and definitions and get reliable email support from our Test Prep Educator and Attorney Elliot Gold.

OUR BAR EXAM PREP TEST INCLUDES...

  • 1,000 MBE Test Questions - 5 Tests
  • Take Unlimited MBE Tests - 1 Year
  • Study Mode & Timed Exam Mode
  • Pause Exams & Continue Later
  • Answers with Detailed Explanations
  • Covering All 7 Areas of Law - 2024
  • Bar Exam Definition Flashcards
  • Master 200 Legal Issues
  • Reliable Support From an Attorney
  • Study On-The-Go On All Devices

Can't find the state test you are looking for?

We have practice tests available for all 50 states. Simply click on the button to see our full list of state specific MBE prep test questions that are available.

Quiz-summary

0 of 10 questions completed

Information

The Multistate Bar Examination (MBE) is a six-hour, 200 question multiple-choice examination covering civil procedure, contracts, torts, constitutional law, criminal law, evidence, and real property. Each exam is carefully crafted to meet or exceed the difficulty level of the actual Multistate Bar Exam.

You have already completed the quiz before. Hence you can not start it again.

Quiz is loading...

You must sign in or sign up to start the quiz.

You have to finish following quiz, to start this quiz:

0 of 10 questions answered correctly

Time has expired!

You have reached 0 of 0 points, ( 0 )

Score breakdown by Topic

We're sorry..., bar exam prep test free trial.

Answered Questions

Herb is selling his home in the northwest to retire in the south. He has a large house filled with antique furniture, but he cannot take it all to his smaller home in the south. The buyer of the home wants it vacant. Herb calls his nephew Joe and tells Joe that if he moves the furniture out of his home, he can have it. Joe agrees, so he rents a truck and drives 300 miles to his uncle’s home and places the furniture into the truck. Before Joe departs, Uncle Herb decides that he wants to keep the furniture. Was a valid contract formed? I. Yes. There was a bargained for exchange. II. Yes. Herb made a valid unilateral contract offer. III. No. Herb merely stated a condition for Joe to receive a gift.

  • b) III only.
  • c) I and II.
  • d) II only.

Answer: a) When Joe agreed to come to Herb’s house, a bargained for exchange was agreed to. Herb was not merely giving Joe a gift, since he needed the furniture removed and Joe was required to rent a truck and travel a long distance to do so. Thus, Herb was bargaining for consideration, which was Joe’s agreement to come take the furniture out of Herb’s home. Herb did not make a unilateral contract offer, which would require performance in order to accept. Joe did not need to completely perform the contract in order to accept.

Question: 2 - 3

Thomas plans on bombing the City marathon the next day with his brother James, because he does not like the way his family is treated. They go into a neighborhood store and discuss the plan with storekeeper Antoine. Antoine is upset with the government for deporting his son and tells the brothers that they should use pressure cooker bombs and sells them the items they need. Antoine charges a higher than usual price because of the circumstances. The brothers successfully execute their plan, the bomb explodes and deaths and injuries result. They return to Antoine’s store and hide out in the basement while the police search the area. The police arrest the brothers, along with Antoine.

Question ##: Answer: c) Accessory before the fact existed in common law, but not in modern statutes. All parties to the crime can be charged for the underlying offenses, so Antoine, as an accomplice to the crimes can be found guilty of battery and murder, since he intentionally assisted and encouraged the commission of the crimes. He is also guilty of Accessory after the Fact, for helping the brothers evade arrest. A conspiracy is also present due to the agreement to commit the bombing.

Question ##: Answer: c) Antoine repudiated his encouragement of the crimes and neutralized his assistance and rendered it ineffective by taking back the materials he sold to the brothers before the crime was committed. Said steps have been found to be sufficient, without having to contact the authorities.

Boris is walking on a County street. His doctor had recommended that he stay in bed. He turns ill and stumbles along the sidewalk. He comes to a sidewalk vault that is flush with the sidewalk, but not in a hazardous condition. Boris misses a step and trips over the vault. He falls to the ground and breaks his left ankle and hip. He sues the owner of the vault for negligently maintaining it. A settlement offer is made to Boris by the vault owner’s insurance company, but Boris rejects it and wants more. At trial, the vault owner’s defense is that Boris was negligent and the sole proximate cause of his injuries. The vault owner calls a man who saw the accident to the stand. The vault owner’s attorney wants to question the witness about what he observed. The attorney asks the witness to describe how Boris looked. The witness states that Boris looked old, ill and looked like he suffered from AIDS. Boris’ attorney objects. Will the Court overrule the objection and allow the witness’ answer to stand?

  • a) Yes, since the witness can testify to a personal observation.
  • b) Yes. The testimony is based upon the witness’ perception.
  • c) The Court will partially overrule the objection, and only allow the testimony pertaining Boris’ age and that he appeared ill.
  • d) No. The objection will be sustained.

Answer: c) The lay witness cannot testify as to a specific illness, but can provide his rationally based perception of the witness if it is helpful for a clear understanding of the testimony, but cannot be based upon scientific, technical or other specialized knowledge. Thus, the witness is permitted to give an overall impression of what was observed.

Milo is out partying with some old college friends. They stay in the bar until closing time. Milo is extremely intoxicated. The bartender offers to call Milo a taxi, but Milo refuses and gets into his car to drive home. Several blocks later, Milo is pulled over by the police. Based upon the officer’s observations of signs of intoxication, he asks Milo to step out of his vehicle. Milo stumbles out of the vehicle and falls to the pavement. The officer helps Milo up and asks Milo if he consumed any alcohol. Milo denies it. Milo is given a sobriety test and fails. He refuses to take a breathalyzer. Milo is arrested based upon suspicion of drunken driving. Since Milo refused a breathalyzer, Milo’s driver’s license is suspended for 12 months and a post-suspension hearing affirms the suspension. Based upon a procedural issue, no charges are brought against Milo, but the suspension remains. Does the suspension run afoul of Milo’s Due Process rights?

  • a) Yes. However, the general public’s need for safe roads meets the strict scrutiny and rational relationship tests.
  • b) Yes. Since no charges were brought against Milo, his license was improperly suspended.
  • c) No. A post-suspension hearing was conducted.
  • d) No. Statutes permitting automatic suspension of a driver’s license are constitutional per se .

Answer: c) While a hearing is generally required before a state can suspend an individual’s driver’s license, the Supreme Court has held that a state’s suspension of a driver’s license does not violate Due Process, so long as a post-suspension hearing is conducted. In the instant case, since a post-suspension hearing was conducted, there was no violation of Milo’s right to Due Process.

Question: 6 - 7

Axton and Valentino are lifelong friends. Axton has had a successful career mining gold from Tombstone, Axton’s 20 acre property, which also has a 4 bedroom home. Valentino has always struggled to make a living. Feeling bad for Valentino, Axton leaves his 20 acre property to Valentino in his will for the life of Valentino and then to Axton’s heirs. When Axton dies, his heirs seek to enjoin Valentino from using the property to mine gold so Valentino does not exploit the natural resources.

Question ##: Will the heirs be able to succeed under the Doctrine of Waste?

Question ##: Answer: c) Since a life estate was left to Valentino, and prior to the grant, the land was used in exploitation of the gold, Axton most likely wanted Valentino to mine for gold.

Question ##: If Axton’s heirs permit Valentino to mine, and the jurisdiction in which the land is located does not follow the Open Mines Doctrine, what are Valentino’s rights?

Question ##: Answer: d) There is a limit with respect to the amount of gold that life estate holder Valentino can mine. The exploitation must be reasonable. The Open Mines Doctrine would restrict Valentino to mining the mines that were previously open before Valentino was left the life estate. Since the doctrine is not in effect, Valentino can mine any part of the property and is not limited to open mines. Since the land was already exploited for gold, Valentino is not restricted to mining solely to pay for repair and maintenance of the property.

Question: 8 - 9

Longtime friends Cary and Brutus are shopping in a clothing store. They are talking about the boxing match they were watching the night before and begin to playfully box with each other in a nonviolent manner by pretending to contact each other, but do not make any contact. Bebe walks by the pair, but does not notice them. As Cary tries to punch Brutus, he moves out of the way and Bebe is lightly struck in the head causing a head injury.

Question ##: Which of the following statements are true? I. Cary has committed an assault against Brutus. II. Cary has committed an assault against Bebe. III. Cary has committed a battery against Bebe. IV. Cary has not committed an intentional tort.

Question ##: Answer: d) A battery was committed against Bebe. A battery requires an offensive or harmful contact, intent and causation . Even though Cary did not intend to strike Bebe, if the intention to commit an intentional tort against Brutus unintentionally resulted in a tort against a third person, Bebe, a battery against the third person results, based upon the transferred intent doctrine . Apprehension of the imminent contact is not necessary. However, no assault has occurred. An assault requires a reasonable apprehension of an immediate harmful or offensive contact to the person, intent to bring about such apprehension and causation. Brutus knew that the fighting was playful and non-contact, so there was no assault. Bebe did not even notice Cary and Brutus, so there was never any apprehension on her part.

Question ##: If Cary did not make contact with Bebe, would an intentional tort still have been committed by Cary?

Question ##: Answer: b) No battery would result to Bebe based upon transferred intent and no assault since Bebe did not even notice Cary and Brutus, so there was never any apprehension on her part.

Question: 10 - 11

Chase is a deadbeat parent, who has evaded paying child support for years. He resides in South Dakota, though he previously resided in North Dakota, where he had lived with his ex-wife Naomi. Chase is aware that Naomi is trying to locate him to serve him with a suit over missed child support payments, so he flees to a Wyoming motel. Naomi has tracked Chase to the motel and she has him personally served there pursuant to a North Dakota statute permitting out of state service on current or prior residents evading child support. Chase remains in Wyoming and hires a lawyer in North Dakota to move to dismiss the case for lack of personal jurisdiction. The attorney argues that the service did not enable jurisdiction over Chase, but offers no additional arguments.

Question ##: Will the Court grant the motion?

Question ##: Answer: a) Chase was domiciled and resided in South Dakota. While the Supreme Court has held that “domicile in the state is alone sufficient to bring an absent defendant within the reach of the state’s jurisdiction for purposes of a personal judgment…”, as long as there is sufficient notice of the proceedings. [Milliken v. Meyer, 311 U.S. 457 (1940)], Chase was no longer a resident nor domiciled in North Dakota.

Question ##: If Naomi brought her case in federal court and personal jurisdiction was obtained, would the result differ?

Question ##: Answer: b) The federal courts do not have subject matter jurisdiction to hear cases involving family/domestic relations law, such as child support, divorce, alimony, etc, so they would dismiss the case for lack of subject matter jurisdiction.

While Junior is out trapping in the Alaskan forest, his neighbor Franco trespasses onto Junior’s land and cuts down 500 of Junior’s corn stalks, which Franco’s wife will use to make corn bread for the county fair. Franco takes 250 of the stalks home and leaves the remaining stalks next to Junior’s barn. Franco plans to come back later at night to pick up the rest. Junior comes home and finds the 250 stalks and is baffled. He moves them onto his flat bed trailer and places a tarp over them. When Franco returns to Junior’s property later that night, he learns that the stalks were moved from the area where he left them next to the barn. He spends a half hour looking for them and finally finds them on Junior’s trailer. He takes off the tarp and brings the stalks home. Junior reviews his video surveillance system and observes Franco cutting down the stalks and his subsequent actions. He contacts the police, who charge Franco with larceny of 500 stalks. Franco objects to the charges. Has Franco committed larceny?

  • a) Yes. He is guilty of larceny of 500 stalks.
  • b) No. He has not committed larceny, only trespass.
  • c) Yes. He is guilty of larceny.
  • d) No. He is guilty of common law conversion.

Answer: c) Larceny cannot arise from taking fixtures detached from the property before it comes into possession of its owner. However, since Franco took possession of the 250 stalks that Junior made his own personal property, larceny of the those stalks was committed.

Summer is an avid fan of Heschitt paintings. She finds one in poor condition at an art sale. She purchases it for $22,000, but knows that it is worth $55,000 in perfect condition. Summer takes the painting to Cooper’s Painting Restoration Company, which specializes in restorations of old paintings. After negotiations, Cooper agrees to restore the painting for $18,000. After working on the painting for 2 weeks, and spending $15,000 on materials, Cooper tells Summer that they will need to charge $12,000 more to make a profit. Summer verbally agrees to the additional cost, since she does not want to bring the painting to an inferior restorer. Cooper completes the painting, but Summer refuses to pay anything more than $18,000. Can Cooper recover the additional $12,000?

  • a) Yes. There was mutual assent to the new contract terms, so the additional cost is due.
  • b) No, unless it is in writing.
  • c) No, based upon a consideration issue.
  • d) Yes, since Summer will still realize a profit if she sells the painting.

Answer: c) The change is not enforceable. Cooper was under a preexisting duty to restore the painting and offered no additional consideration in exchange for the additional cost.

Question: 14 - 15

Maridale, an adult male, was raised in the jungles of Zimbabgo and lived in caves. He has no access to modern conveniences like electric or automobiles and he has never used any fuels. He is being discriminated against by government officials and applies to move to the United States as part of a refugee program. He is brought to the U.S. and is asked to take English classes. While at a class inside a school, he ventures outside and finds the local zoo and wanders inside. He had never seen a zoo before and does not understand the concept. He decides to free the animals and walks to the bull’s den and opens up the gate. He then continues to walk through the zoo. The bull eventually gets out of the enclosure. An unsuspecting zoo visitor sees the unrestrained bull and screams. The bull is startled and charges towards the visitor and impales her with his horns. The zoo visitor sues the zoo and Maridale.

Question ##: If Maridale’s attorney argues that Maridale had never seen a zoo before and did not know the dangers of letting a bull free, will the attorney’s argument succeed?

Question ##: Answer: a) Maridale is presumed to possess the same knowledge that the average member of the community would possess. The average community member should know that a bull on the loose is extremely dangerous and could cause serious injury or death.

Question ##: If instead of being raised in the jungle, Maridale lived in a regular community, but was mentally handicapped, which prevented him from understanding the consequences of his actions, would the result change?

Question ##: Answer: c) Maridale’s individual mental insufficiencies do not matter. He must act as a reasonable person would. Individual mental handicaps are not considered when using the reasonable person standard.

Already a Member?

Banner

Bar Exam Resources: New York

  • Practice Exams and Books
  • Multistate Exams and Other States
  • Uniform Bar Exam
  • Commercial Bar Exam Courses

Free MBE Questions

  • Bar Prep Hero Provides a review and preparation program that deals with the MBE part of the test, which covers all testing administered on day one of the bar examination.

NY Law Exam

Among other requirements , candidates for admission to the New York bar must take the New York Law Course (NYLC) and the New York Law Exam (NYLE).

The NYLC is a prerequisite to the NYLE.   More information is available on the BOLE website .

The NYLE will be offered four times per year. The dates and times of administration of the NYLE in 2019 are:

  • March 21, 2019, 12:00 pm EST
  • June 13, 2019, 12:00 pm EST
  • Sept. 26, 2019, 12:00 pm EST
  • Dec. 19, 2019, 12:00 pm EST
  • Information about Course Materials for NYLC and NYLE
  • Course Materials for the NYLC/NYLE
  • Sample Questions for the NYLE
  • Introduction to New York Law Course "[NYLC] is an online, on demand course on important and unique principles of New York law in the following subjects: Administrative Law, Business Relationships, Civil Practice and Procedure, Conflict of Laws, Contracts, Criminal Law and Procedure, Evidence, Matrimonial and Family Law, Professional Responsibility, Real Property, Torts and Tort Damages, and Trusts, Wills and Estates. Completion of the NYLC is required in order to obtain a license to practice law in the State of New York."

General Information

  • New York State Board of Law Examiners (BOLE) Information about applying for the bar, bar exam results, old bar exam questions and answers.
  • Advisory Committee on the Uniform Bar Examination "The Advisory Committee on the Uniform Bar Examination (UBE) was created by Chief Judge Jonathan Lippman to comprehensively examine a proposal to adopt the UBE as part of the New York bar examination. A study committee chaired by the Honorable Jenny Rivera, Associate Judge of the New York State Court of Appeals, and comprised of representatives of law schools, the bar, the judiciary and the State Board of Law Examiners, is considering this proposal."
  • Preparing for the Bar Exam Advice from the Academic Support Program at Pace Law School.
  • Reports and Press Releases from NY BOLE Press releases, bar passage rate statistics, and reports from the BOLE.
  • Bar Exam Results (BOLE) Alphabetical list of candidates who passed the bar.
  • Which Bar Prep Course is the Best? Brian Dalton provides information about the major bar prep providers for Above the Law. He analyzes variables including cost, format, guarantees, discounts, and pass rate. Keep in mind that you can sometimes negotiate a better price than what is listed.
  • Developing Legal Careers and Delivering Justice in the 21st Century Report by the NYC Bar Association Task Force on New Lawyers in a Changing Profession, issued Fall 2013. Discusses changes to the NY bar exam to reflect the needs of the profession, including dropping UCC article 3 (negotiable instruments) in July 2014 and adding administrative law in February 2015.

New Book on Reserve

new york state bar exam practice questions

This guide was created by Cynthia Pittson and is currently maintained by Pace Law Library Reference Librarians.

Description of UBE Administered in NY

From the BOLE website :

The UBE is administered on the last Tuesday and Wednesday of February and July.

During the morning session on Tuesday , applicants are given three hours to complete two Multistate Performance Test (MPT) items. Applicants may work on the MPTs in any order and they are free to decide how to allocate their time between the two MPT items although NCBE develops each MPT as a 90 minute test item.

During the afternoon session on Tuesday , applicants are given three hours to answer six Multistate Essay Exam (MEE) questions. Again, applicants are free to answer the questions in any order and they may decide how to allocate their time among the MEE questions.

On Wednesday , applicants will take the Multistate Bar Examination (MBE), which is a six-hour, 200 question multiple-choice exam divided into two three-hour sessions.

The schedule for the UBE in New York is as follows:

  • Tuesday: 9:30 am to 12:30 pm and 2:00 pm to 5:00 pm
  • Wednesday: 9:30 am to 12:30 pm and 2:00 pm to 5:00 pm

The July 2019 bar exam is scheduled for the last Tuesday and Wednesday of the month, July 30-31, 2019.

  • MEE (essays) = 30% of score

Passing score for NY on the UBE is 266.

MBE, MEE, and MPT Multistate Exams

  • MBE Information about the Multistate Bar Examination, a six-hour, 200-question multiple-choice examination covering contracts, torts, constitutional law, criminal law and procedure, evidence, and real property.
  • MBE Preparation Includes sample test questions.
  • MBE Subject Matter Outline Updated for 2020
  • MEE Multistate Essay Exam, part of the UBE.
  • MEE Preparation Includes sample test questions.
  • MEE Subject Matter Outline Updated for 2019.
  • MPT The Multistate Performance Test consists of two 90-minute skills questions covering legal analysis, fact analysis, problem solving, resolution of ethical dilemmas, organization and management of a lawyering task, and communication.
  • MPT Summaries 2012-2018 Provides a description of the MPT, an outline of the skills tested, and summaries of previously administered tests.
  • Questions & Sample Answers 2004-2018, NY Bar Examiners
  • Sample MPT & MEE Questions with Answers From NJ Board of Bar Examiners
  • Sample MPT & MEE Questions with Answers (July 2019) From NJ Board of Bar Examiners
  • MPT Feb. 1998
  • MPT July 1998

Nuts and Bolts of the NY Bar Exam

  • Next: Practice Exams and Books >>
  • Last Updated: Jul 28, 2023 1:28 PM
  • URL: https://libraryguides.law.pace.edu/barexam

  office (518) 453-5990 fax (518) 452-5729 TTY: Call 711 (nyrelay.com) Phone Hours: Monday - Friday 8:30am – 4:30pm Eastern Time

  • BOLE Account
  • UBE / NYLC / NYLE FAQ's

Admission Information

  • Application by Transferred UBE Score
  • Bar Exam Eligibility
  • Bar Exam Information
  • Bar Exam Statistics
  • Change of Address
  • Dates of Exams and Deadlines
  • Exam Day References & Security Policy
  • Foreign Legal Education
  • Mandatory 50-Hour Pro Bono Rule
  • Misconduct Policy
  • NYLC / NYLE Course Access & Materials
  • Other Links & Contact Information
  • Past Bar Exam Questions & Answers
  • Past Bar Exam Results
  • Press Releases and Reports
  • Pro Bono Scholars Program
  • Proctor Information
  • Reciprocity / Motion Information
  • Rules and Regulations
  • Skills Competency Requirement for Bar Admission
  • Study Guides for UBE and MPRE
  • Test Accommodations

NOTICE FOR SPRING 2020 GRADUATES REGARDING APPLICATION FOR ADMISSION AND ADMISSION REQUIREMENTS: Click here to access the APRIL 27, 2020 STATEMENT FROM THE APPELLATE DIVISION PRESIDING JUSTICES REGARDING BAR ADMISSION PRACTICES.

Multi Department Admission Application

Admissions, Swearing In, and Character and Fitness: Any questions on admission to the Bar, swearing in and character and fitness application materials should be directed to the appropriate Appellate Division of the Supreme Court. Each applicant for admission should visit the website of the appropriate Appellate Division of the Supreme Court by clicking on the link in the box below to learn of their individual requirements or forms. The telephone numbers and website links are as follows:

First department (646) 386-5893 second department (718) 923-6360 third department (518) 471-4778 fourth department (585) 530-3100 the board of law examiners has no information about, and is unable to answer questions regarding the application for admission to practice, which you may download by clicking the link above "multi-department admission packet." you should read the general instructions very carefully, and direct any questions to the judicial department in which you reside or to which you have been certified. if you change your address before you receive your results from the bar examination, you must immediately notify the board of such change using the board’s online change of address request form found here . changes which affect certification to the appellate division for admission purposes must be received by the board by october 15 for a july exam and april 15 for a february exam. once you are certified for admission, any change of address must be made through the appellate division to which you were certified..

Section 520.12(d)(1) of the Rules of the New York State Court of Appeals for the Admission of Attorneys and Counselors at Law requires applicants to file the complete application for admission with the Supreme Court, Appellate Division within three (3) years from the date when the applicant sat for the second day of the Uniform Bar Examination, whether taken in New York or in another jurisdiction. Pursuant to Section 520.12(d)(2) applicants who passed the bar examination administered prior to July 2016 must file the complete application for admission within three (3) years from the date of the letter sent by the Board notifying the applicant that the applicant passed the bar examination. A failure to timely file the application for admission may result in the applicant having to re-sit the bar examination.

PRO BONO REQUIREMENT

On September 14, 2012, the New York State Court of Appeals adopted a new rule requiring applicants who successfully pass the bar examination in New York State to perform 50 hours of pro bono services before applying for admission to practice. Applicants seeking admission to practice after January 1, 2015, must demonstrate compliance with this requirement. Please note that this requirement does NOT apply to applicants seeking admission on motion pursuant to Rule 520.10. For more information, please see http://www.nycourts.gov/attorneys/probono/baradmissionreqs.shtml

  Contact US | Disclaimer

Links to other sites, or links to this site by any other sites, do not imply any endorsement of, or relationship with, such other sites

  • MPRE Registration Guide
  • MPRE Study Guide
  • Bar Exam Tips and Information
  • Premium Login
  • Pass the first time, guaranteed
  • Help Center

Breeze through your Bar Exam, guaranteed

Bar exam prep is overdue for an upgrade. This is it. Skyrocket your chances of passing with 1,300+ real MBE questions licensed from NCBE, plus over 600 simulated MBE questions, and Bar Exam Genie AI - a first-of-its-kind tutor.

  • Connecticut
  • District Columbia
  • Massachusetts
  • Mississippi
  • New Hampshire
  • North Carolina
  • North Dakota
  • Pennsylvania
  • Rhode Island
  • South Carolina
  • South Dakota
  • West Virginia

Get your hands on the last 23 years of official MEE essays - with full analyses. Plus the last 23 years of MPT tests, complete with the legal points, facts and possible issues within the lawyering tasks described.

Welcome to Barprephero Flashcards! We now have another proven way to help you study for your Bar examination. Scientific research shows that using flashcards as a study method creates strong neural brain connections. This means your memory improves as you go through the flashcards again and again; this is called "active recall." You’ll also gain confidence as you realize that you know and understand the information. Flashcards are one of the most effective study tools, because you know instantly if you have the correct answer. Since you’re "grading your own work," you will learn more quickly and be able to focus on areas that need further study.

  • Single-Subject MBE Tests
  • Mixed-Subject MBE Tests

MBE Exam Simulator

Con law test 1.

This demo prep test includes 15 simulated questions to familiarize you with the kinds of Constitutional Law questions that may appear on the MBE.

Con Law Test 2

This test includes 211 official Constitutional Law questions. The terms “Constitution,” “constitutional,” and “unconstitutional” refer to the federal Constitution unless indicated otherwise.

Con Law Marathon

Includes all 312 Constitutional Law questions (a mix of both simulated and official questions from past exams). If you make a mistake, we’ll show that question to you again until you’ve answered it correctly.

What’s Your Time and Sanity Worth?

Most traditional bar review programs are priced ridiculously high, yet they offer limited access (only unlock after a certain date) and fewer practice materials. See why BarPrepHero guarantees you’ll pass this year’s Bar exam with confidence -- without breaking the bank.

Contracts Test 1

15 demo questions on Contract Law. Revised Articles 1 and 2 of the Uniform Commercial Code are applicable to merchants/sale of goods where appropriate.

Contracts Test 2

190 officially licensed MBE questions on Contract Law drawn from past exams. Examinees are to assume that the Official Text of Articles 1 and 2 of the Uniform Commercial Code have been adopted and is in effect.

Contracts Marathon

Includes all 297 Contracts questions (a mix of both simulated and official questions from past exams). If you make a mistake, we’ll show that question to you again until you’ve answered it correctly.

Crim Law Test 1

This demo prep test gives you practice on the types of questions contained in the Multistate Bar Exam with respect to Criminal Law and Procedure. It is the usual multiple-choice format with four possible answers. You are to choose the best answer of the four.

Crim Law Test 2

This test lets you practice 193 real MBE questions on Criminal Law and Procedure. Approximately half of the Criminal Law and Procedure questions on the MBE will be based on category V, and approximately half will be based on the remaining categories – I, II, III, and IV.

Crim Law Marathon

Includes all 311 Criminal Law and Procedure questions (a mix of both simulated and official questions from past exams). If you make a mistake, we’ll show that question to you again until you’ve answered it correctly.

Evidence Test 1

This 98-question test deals with the subject of Evidence. You will be able to familiarize yourself with the types of questions on this topic that you may encounter on the official MBE.

Evidence Test 2

This test deals with the subject of Evidence and contains 207 official MBE questions from past exams. All Evidence questions should be answered according to the Federal Rules of Evidence, as currently in effect.

Evidence Marathon

Includes all 305 Evidence questions (a mix of both simulated and official questions from past exams). If you make a mistake, we’ll show that question to you again until you’ve answered it correctly.

Real Property 1

This is the practice examination for Real Property. It will assess how well you can apply fundamental legal principles and legal reasoning to analyze problems in this practice area.

Real Property 2

This is the practice examination with official questions for Real Property. It will assess how well you can apply fundamental legal principles and legal reasoning to analyze problems in this practice area.

Real Property Marathon

Includes all 304 Real Property questions (a mix of both simulated and official questions from past exams). If you make a mistake, we’ll show that question to you again until you’ve answered it correctly.

Torts Test 1

This is the practice examination for Torts. The bar exam questions should be answered based on principles of general applicability.

Torts Test 2

This is the practice examination for Torts. The 217 official Multistate Bar Exam questions should be answered based on principles of general applicability. Examinees are to assume that survival actions and claims for wrongful death are available.

Torts Marathon

Includes all 322 Torts questions (a mix of both simulated and official questions from past exams). If you make a mistake, we’ll show that question to you again until you’ve answered it correctly.

Civil Procedure 1

This 132-question prep test deals with Civil Procedure. This topic is a recent addition to the topics covered on the MBE. The National Conference of Bar Examiners advises you to answer the Civil Procedure questions according to accepted fundamental legal principles.

Civil Procedure 2

This test contains official Civil Procedure MBE questions. Examinees are to assume the application of (1) the Federal Rules of Civil Procedure as currently in effect and (2) the sections of Title 28 of the U.S. Code pertaining to trial and appellate jurisdiction, venue, and transfer.

Civil Procedure Marathon

Includes all 219 Civil Procedure questions (a mix of both simulated and official questions from past exams). If you make a mistake, we’ll show that question to you again until you’ve answered it correctly.

Practice Exam 1

This practice test covers the following legal practice areas: Civil Procedure, Constitutional Law, Contracts, Criminal Law and Procedure, Evidence, Real Property, and Torts.

Practice Exam 2

This practice test combines material from most of the seven legal practice areas covered in the MBE. It should be similar to the official MBE.

Practice Exam 3

The questions on this test are based on a mixture of most of the seven subject areas in the official MBE.

Practice Exam 4

This is the fourth prep test to assist you in preparing for the MBE. The test contains 100 multiple-choice questions that are similar to those on the actual MBE.

Practice Exam 5

The fifth prep test contains 100 mixed subject matter questions.

Practice Exam 6

Our sixth prep test contains 150 questions based on the types of questions on the official MBE.

Practice Exam 7

Our seventh prep test contains 160 questions based on the types of questions on the official MBE.

This annotated practice exam uses questions drawn from actual MBEs administered in recent years and officially licensed from NCBE. The annotations explain why each answer choice is correct or incorrect. Six MBE subject areas: Constitutional Law, Contracts, Criminal Law and Procedure, Evidence, Real Property, and Torts.

Practice Exam 8

Practice exam 9.

This practice exam consists of 100 official questions released by the NCBE that appeared in previously administered MBEs. Does not contain Civil Procedure questions. The purpose of this exam is to familiarize you with the format and nature of MBE questions.

Practice Exam 10

Practice exam 11, practice exam 12, practice exam 13.

This practice exam consists of 100 official questions released by the NCBE that appeared in previously administered MBEs. The purpose of this exam is to familiarize you with the format and nature of MBE questions.

Practice Exam 14

This 200-question exam simulates the actual MBE and contains a mix of questions from our other tests on the following subjects: Contracts, Torts, Constitutional Law, Criminal Law and Procedure, Evidence, Real Property, and Civil Procedure. You’re allowed to skip questions, but unlike the other practice tests, this test simulator provides no hints or explanations.

  • MEE (Essays)

2023 Questions & Analyses

new york state bar exam practice questions

2022 Questions & Analyses

2021 questions & analyses, 2020 questions & analyses, 2019 questions & analyses, 2018 questions & analyses, 2017 questions & analyses, 2016 questions & analyses, 2015 questions & analyses, 2014 questions & analyses, 2013 questions & analyses, 2012 questions & analyses, 2011 questions & analyses, 2010 questions & analyses, 2009 questions & analyses, 2008 questions & analyses, 2007 questions & analyses, 2006 questions & analyses, 2005 questions & analyses, 2004 questions & analyses, 2003 questions & analyses, 2002 questions & analyses, 2001 questions & analyses, 2000 questions & analyses, 1999 questions & analyses, 1998 questions & analyses, 1997 questions & analyses, 2023 tests & point sheets.

new york state bar exam practice questions

2022 Tests & Point Sheets

2021 tests & point sheets, 2020 tests & point sheets, 2019 tests & point sheets, 2018 tests & point sheets, 2017 tests & point sheets, 2016 tests & point sheets, 2015 tests & point sheets, 2014 tests & point sheets, 2013 tests & point sheets, 2012 tests & point sheets, 2011 tests & point sheets, 2010 tests & point sheets, 2009 tests & point sheets, 2008 tests & point sheets, 2007 tests & point sheets, 2006 tests & point sheets, 2005 tests & point sheets, 2004 tests & point sheets, 2003 tests & point sheets, 2002 tests & point sheets, 2001 tests & point sheets, 2000 tests & point sheets, 1999 tests & point sheets, 1998 tests & point sheets, 1997 tests & point sheets.

  • 0% proficiency

Criminal Law

Real property, constitutional law, civil procedure, security transactions and mortgages, wills, trusts and estates, business organizations.

Glad I found this site. The explanations and cited cases is a huge plus. I think that they are good sharpening analytical skills. Keep up the good work here! ~ Jacqueline
First allow me to say that you guys are awesome, and I greatly appreciate your bar-prep site. I am currently a paralegal student, and have always wanted to know how I would perform on the bar multi choice section. Your site has given me that opportunity, and I am pleasantly surprised with most of my test scores (70’s -- 90’s), depending on whether I have had a course in a particular test area. Even mistaken answers become learning tools due to your highly instructive and nicely detailed answer sections. I have recommended your site to an attorney who has moved here from another state, and is now preparing for the Maryland Bar Exam. I can not thank you guys enough for your efforts in creating your site, I am very pleased with the level of confidence I have found in myself, by getting the number of correct answers as I have so far. Always wanted to know, and you guys are providing the challenge. Thanks, and please keep the questions coming. Best Regards, Bill Bennett ~ William Bennett, Maryland
This was very helpful. One of my classmates recommended this to me. He got 100% on the practice MBE’s in class last night and credits this page for it. ~ Ellen Fawl, California
Barprephero is fantastic. This gave me the inspiration to further my research in law. Thanks! ~ Edwin Vargas
I’ve written several books on bar preparation in my “In-Brief” series currently selling on Amazon. While writing the books, I received advice from a psychologist who told me that stories are easier to remember than rules. So for each tested bar topic, I included a case that reflects a story relating to the rule. BarPrepHero is the only test site that I have participated in which explains virtually all of its answers with a related case, and as such provides a mechanism for retaining rules through stories. Five stars in my book! ~ Glenn Riser, Juris Doctor Graduate at Florida Coastal School of Law
This is a great introductory MBE study tool. I have taken and passed the California bar exam previously and I am studying for the Tennessee bar exam currently. I feel that the questions provided are not as difficult as the actual MBE questions on the exam, but they are an excellent refresher and a wonderful study aid. ~ Chrystine Carvalho, TN
Some of these I recognize from actual cases I read in Crim Law!! Love it! ~ Jessica Stone-Erdman, NC
Fantastic prep for Evidence final. Thank you! ~ Anton Yatsenko
You know you’ve crossed the line into full on bar prep nerddom when you get excited about finding free MBE practice! ~ Susan Zucco

What is taking the Bar Exam really like?

Bar admission process: how the bar exam is administered, bar exam day one: the multistate bar examination (mbe), bar exam day two: state-specific law questions, pass the next bar exam, guaranteed, constitutional law subject matter outline.

NOTE: The terms "Constitution," "constitutional," and "unconstitutional" refer to the federal Constitution unless indicated otherwise. Approximately half of the Constitutional Law questions on the MBE will be based on category IV, and approximately half will be based on the remaining categories - I, II, and III.

  • Organization and relationship of state and federal courts in a federal system
  • Congressional power to define and limit
  • The Eleventh Amendment and state sovereign immunity
  • The "case or controversy" requirement, including the prohibition on advisory opinions, standing, ripeness, and mootness
  • The "adequate and independent state ground"
  • Political questions and justiciability
  • Commerce, taxing, and spending powers
  • War, defense, and foreign affairs powers
  • Power to enforce the 13th, 14th, and 15th Amendments
  • Other powers
  • As chief executive, including the "take care" clause
  • As commander in chief
  • Treaty and foreign affairs powers
  • Appointment and removal of officials
  • Congressional limits on the executive
  • The presentment requirement and the president's power to veto or to withhold action
  • Nondelegation doctrine
  • Executive, legislative, and judicial immunities
  • Federal immunity from state law
  • State immunity from federal law, including the 10th Amendment
  • Negative implications of the commerce clause
  • Supremacy clause and preemption
  • Authorization of otherwise invalid state action
  • State action
  • Fundamental rights
  • Other rights and interests
  • Procedural due process
  • Classifications subject to heightened scrutiny
  • Rational basis review
  • Other protections, including the privileges and immunities clauses, the contracts clause, unconstitutional conditions, bills of attainder, and ex post facto laws
  • Free exercise
  • Establishment
  • Content-based regulation of protected expression
  • Content-neutral regulation of protected expression
  • Regulation of unprotected expression
  • Regulation of commercial speech
  • Regulation of, or impositions upon, public school students, public employment, licenses, or benefits based upon exercise of expressive or associational rights
  • Regulation of expressive conduct
  • Prior restraint, vagueness, and overbreadth
  • Freedom of the press
  • Freedom of association

Contracts Subject Matter Outline

NOTE: Examinees are to assume that the Official Text of Articles 1 and 2 of the Uniform Commercial Code have been adopted and is in effect. Approximately half of the Contracts questions on the MBE will be based on categories I and IV, and approximately half will be based on the remaining categories - II, III, V, and VI. Approximately one-fourth of the Contracts questions on the MBE will be based on the Official Text of Articles 1 and 2 of the Uniform Commercial Code.

  • Mutual assent (including offer and acceptance, unilateral, bilateral, and implied-in-fact contracts)
  • Indefiniteness and absence of terms
  • Consideration (bargained-for exchange)
  • Obligations enforceable without a bargained-for exchange (including reliance and restitution)
  • Modification of contracts
  • Incapacity to contract
  • Duress and undue influence
  • Mistake and misunderstanding
  • Fraud, misrepresentation, and nondisclosure
  • Illegality, unconscionability, and public policy
  • Statute of frauds
  • Parol evidence
  • Interpretation
  • Omitted and implied terms
  • Conditions (express and constructive)
  • Excuse of conditions
  • Breach (including material and partial breach, and anticipatory repudiation)
  • Obligation of good faith and fair dealing
  • Express and implied warranties in sale-of-goods contracts
  • Other performance matters (including cure, identification, notice, and risk of loss)
  • Impossibility, impracticability, and frustration of purpose
  • Discharge of duties (including accord and satisfaction, substituted contract, novation, rescission, and release)
  • Expectation interest (including direct, incidental, and consequential damages)
  • Causation, certainty, and foreseeability
  • Liquidated damages and penalties, and limitation of remedies
  • Avoidable consequences and mitigation of damages
  • Rescission and reformation
  • Specific performance and injunction
  • Reliance and restitution interests
  • Remedial rights of breaching parties
  • Third-party beneficiaries
  • Assignment of rights and delegation of duties

Criminal Law and Procedure Subject Matter Outline

NOTE: Approximately half of the Criminal Law and Procedure questions on the MBE will be based on category V, and approximately half will be based on the remaining categories - I, II, III, and IV.

  • Premeditation, deliberation
  • Provocation
  • Intent to injure
  • Reckless and negligent killings
  • Felony murder
  • Misdemeanor manslaughter
  • Theft and receiving stolen goods
  • Assault and battery
  • Rape; statutory rape
  • Possession offenses
  • Solicitation
  • Parties to crime
  • Acts and omissions
  • Required mental state
  • Strict liability
  • Mistake of fact or law
  • Mental disorder
  • Intoxication
  • Justification and excuse
  • Jurisdiction
  • Arrest, search and seizure
  • Confessions and privilege against self-incrimination
  • Lineups and other forms of identification
  • Right to counsel
  • Fair trial and guilty pleas
  • Double jeopardy
  • Cruel and unusual punishment
  • Burdens of proof and persuasion
  • Appeal and error

Evidence Subject Matter Outline

NOTE: All Evidence questions should be answered according to the Federal Rules of Evidence, as currently in effect. Approximately one-quarter of the Evidence questions on the MBE will be based on category I, one-third on category II, one-quarter on category V, and the remainder on categories III and IV.

  • Requirement of personal knowledge
  • Refreshing recollection
  • Objections and offers of proof
  • Lay opinions
  • Competency of witnesses
  • Judicial notice
  • Roles of judge and jury
  • Limited admissibility
  • Presumptions
  • Control by court
  • Scope of examination
  • Form of questions
  • Exclusion of witnesses
  • Inconsistent statements and conduct
  • Bias and interest
  • Conviction of crime
  • Specific instances of conduct
  • Character for truthfulness
  • Ability to observe, remember, or relate accurately
  • Impeachment of hearsay declarants
  • Rehabilitation of impeached witnesses
  • Contradiction
  • Proceedings to which evidence rules apply
  • Exclusion for unfair prejudice, confusion, or waste of time
  • Authentication and identification
  • Admissibility of character
  • Methods of proving character
  • Habit and routine practice
  • Other crimes, acts, transactions, and events
  • Prior sexual misconduct of a defendant
  • Qualifications of witnesses
  • Bases of testimony
  • Ultimate issue rule
  • Reliability and relevancy
  • Proper subject matter for expert testimony
  • Real, demonstrative, and experimental evidence
  • Spousal immunity and marital communications
  • Attorney-client and work product
  • Physician/psychotherapist-patient
  • Other privileges
  • Insurance coverage
  • Remedial measures
  • Compromise, payment of medical expenses, and plea negotiations
  • Past sexual conduct of a victim
  • Requirement of original
  • Completeness rule
  • What is hearsay
  • Prior statements by witness
  • Statements attributable to party-opponent
  • Multiple hearsay
  • Present sense impressions and excited utterances
  • Statements of mental, emotional, or physical condition
  • Statements for purposes of medical diagnosis and treatment
  • Past recollection recorded
  • Business records
  • Public records and reports
  • Learned treatises
  • Former testimony; depositions
  • Statements against interest
  • Other exceptions to the hearsay rule
  • Right to confront witnesses

Real Property Subject Matter Outline

NOTE: Approximately one-fifth of the Real Property questions on the MBE will be based on each of the categories I through V.

  • Fees simple
  • Defeasible fees
  • Life estates
  • Remainders, vested and contingent
  • Executory interests
  • Possibilities of reverter, powers of termination
  • Rules affecting these interests (including survivorship, class gifts, waste, and cy pres)
  • Types: tenancy in common and joint tenancy
  • Relations among cotenants
  • Types of tenancies
  • Possession and rent
  • Transfers by landlord or tenant
  • Termination (including surrender, mitigation of damages, anticipatory breach, and security deposits)
  • Habitability and suitability
  • Rule against perpetuities: common law rule and statutory reforms
  • Alienability, descendibility, and devisability of present and future interests
  • Fair housing/discrimination
  • Conflicts of law related to disputes involving real property
  • Nature and type
  • Termination
  • Property owners’ associations and common interest ownership communities
  • Prescription
  • Scope and apportionment
  • Zoning laws
  • Protection of pre-existing property rights
  • Rezoning and other zoning changes
  • Real estate brokerage
  • Statute of frauds and exceptions
  • Essential terms
  • Time for performance
  • Remedies for breach
  • Marketability of title
  • Equitable conversion (including risk of loss)
  • Options and rights of first refusal
  • Fitness and suitability
  • Purchase money mortgages
  • Future advance mortgages
  • Installment land contracts
  • Absolute deeds as security
  • Necessity and nature of obligation
  • Mortgage theories: title, lien, and intermediate
  • Rights and duties prior to foreclosure
  • Right to redeem and clogging the equity of redemption
  • Assumption and transfer subject to
  • Rights and obligations
  • Application of subrogation and suretyship principles
  • Restrictions on transfer (including due-on-sale clauses)
  • By mortgagee
  • Payment (including prepayment)
  • Deed in lieu of foreclosure
  • Acceleration
  • Parties to the proceeding
  • Deficiency and surplus
  • Redemption after foreclosure
  • Adverse possession
  • Requirements for deed
  • Types of deeds (including covenants for title)
  • Drafting, review, and negotiation of closing documents
  • Persons authorized to execute documents
  • Exoneration
  • Chain of title
  • Hidden risks (e.g., undelivered or forged deed)
  • Title insurance
  • Special problems (including estoppel by deed and judgment and tax liens)

Torts Subject Matter Outline

NOTE: Examinees are to assume that survival actions and claims for wrongful death are available. Joint and several liability, with pure comparative negligence, is the relevant rule unless otherwise indicated. Approximately half of the Torts questions on the MBE will be based on category II, and approximately half will be based on the remaining categories - I, III, and IV.

  • Harms to the person, such as assault, battery, false imprisonment, and infliction of mental distress; and harms to property interests, such as trespass to land and chattels, and conversion
  • Privileges and immunities: protection of self and others; protection of property interests; parental discipline; protection of public interests; necessity; incomplete privilege
  • The duty question, including failure to act, unforeseeable plaintiffs, and obligations to control the conduct of third parties
  • The reasonably prudent person: including children, physically and mentally impaired individuals, professional people, and other special classes
  • Rules of conduct derived from statutes and custom
  • Problems relating to proof of fault, including res ipsa loquitur
  • But for and substantial causes
  • Harms traceable to multiple causes
  • Questions of apportionment of responsibility among multiple tortfeasors, including joint and several liability
  • Problems relating to “remote” or “unforeseeable” causes, “legal” or “proximate” cause, and “superseding” causes
  • Claims against owners and occupiers of land
  • Claims for mental distress not arising from physical harm; other intangible injuries
  • Claims for pure economic loss
  • Employees and other agents
  • Independent contractors and nondelegable duties
  • Contributory fault, including common law contributory negligence and last clear chance, and the various forms of comparative negligence
  • Assumption of risk
  • Strict liability and products liability: common law strict liability, including claims arising from abnormally dangerous activities, and defenses to such claims; claims against manufacturers and other defendants arising out of the manufacture and distribution of products, and defenses to such claims
  • Claims based on nuisance, and defenses
  • Claims based on defamation and invasion of privacy, defenses, and constitutional limitations
  • Claims based on misrepresentations, and defenses
  • Claims based on intentional interference with business relations, and defenses

Civil Procedure Subject Matter Outline

NOTE: Examinees are to assume the application of (1) the Federal Rules of Civil Procedure as currently in effect and (2) the sections of Title 28 of the U.S. Code pertaining to trial and appellate jurisdiction, venue, and transfer. Approximately two-thirds of the Civil Procedure questions on the MBE will be based on categories I, III, and V, and approximately one-third will be based on the remaining categories - II, IV, VI, and VII.

  • Federal subject-matter jurisdiction (federal question, diversity, supplemental, and removal)
  • Personal jurisdiction
  • Service of process and notice
  • Venue, forum non conveniens, and transfer
  • State law in federal court
  • Federal common law
  • Preliminary injunctions and temporary restraining orders
  • Pleadings and amended and supplemental pleadings
  • Joinder of parties and claims (including class actions)
  • Discovery (including e-discovery), disclosure, and sanctions
  • Adjudication without a trial
  • Pretrial conference and order
  • Right to jury trial
  • Selection and composition of juries
  • Requests for and objections to jury instructions
  • Pretrial motions, including motions addressed to face of pleadings, motions to dismiss, and summary judgment motions
  • Motions for judgments as a matter of law (directed verdicts and judgments notwithstanding the verdict)
  • Posttrial motions, including motions for relief from judgment and for new trial
  • Defaults and dismissals
  • Jury verdicts—types and challenges
  • Judicial findings and conclusions
  • Effect; claim and issue preclusion
  • Availability of interlocutory review
  • Final judgment rule
  • Scope of review for judge and jury

Why Study Simulated Questions?

With BarPrepHero Premium, you get access to over 500 exam-like MBE practice questions. Those were carefully crafted by our researchers to ensure they cover the legal areas where we felt the official NCBE questions were not detailed enough.

Together with the real released MBE questions from previous exams, you can truly get the best of both worlds and eliminate surprises on the exam day.

Why Study Authentic MBE Questions?

With BarPrepHero Premium, you get access to all official bar exam questions ever released. The official questions written by the National Conference of Bar Examiners (creators of the Multistate Bar Exam) are far superior to questions invented by commercial courses, and students often see their score improve significantly after using released MBE questions.

Students report that they feel more prepared for the bar exam after using released questions.

Support NYU Law

  • Graduate Affairs
  • Graduate Student Life
  • LLM Student Handbook
  • New York Bar Eligibility

New York Bar Exam Eligibility and Admission to the Bar

§520.6 of the Rules of the Court of Appeals for the Admission of Attorneys and Counselors at Law (22 NYCRR) governs the eligibility of foreign trained lawyers for the New York bar. You will find the full text of §520.6 online. While no one at NYU School of Law can speak on behalf of the New York Court of Appeals, we try to guide students through the process of determining eligibility for the New York bar exam and the subsequent steps to bar admission. 

The information below relates to bar eligibility and admission in New York State, the jurisdiction for which bar admission is most commonly sought by our LLM students and graduates. Those seeking qualification in another state or in multiple states should review the rules in the relevant jurisdiction(s).  The National Conference of Board Examiners provides some information about nearly all of the jurisdictions.  You may also find helpful the information on Disclosures About State Licensing  on the Law School's website.

Policy and Procedure Modifications

There have been modifications to the policies and procedures relevant to the New York bar examination and New York bar admission.  Given the rapidity with which rules can change, please refer frequently to the website of the New York Board of Law Examiners (or to the websites of bar authorities in other jurisdictions you are interested in) to be sure that you have accurate information.

Steps to New York Bar Admission

The following 10 steps are meant to serve as guideposts along the route to New York bar admission.  We advise students to review all of the information in advance of starting the LLM program.  In particular, students should pay attention to Steps 1-4 in the months before they arrive for the LLM program, and complete Step 5 in July (before they depart their home country for their studies at NYU).   

1. Consider whether the NY bar will be useful to you

There are important reasons to reflect on whether taking the NY bar will be helpful to you. For those hoping to practice law in New York for the long-term, NY bar admission is essential. It may still be useful for those looking for short-term employment in New York because some employers look for students planning to take the bar exam as a sign of commitment to the US market. Some students hope to work for a US-based law firm overseas, and the NY bar can be an asset (though not always a requirement) in overseas offices. Many students take the bar because they believe it completes their training as a US lawyer – you've spent the year studying US law, and finalizing your credentials with NY bar admission can complete that package.

On the other hand,  if you need a US LLM for bar eligibility, depending on your curricular interests, meeting eligibility requirements may limit your ability to choose classes. With careful planning, all of our master’s programs provide the flexibility needed to meet both degree requirements and NY bar eligibility requirements. However, only classes in certain areas of law count toward establishing bar eligibility. Flexibility to choose courses most relevant to a practice area may be more important to your career development than NY bar admission.

You may wish also to take into consideration the costs associated with seeking bar admission, such as fees for applying to sit for the exam, the cost of a bar study course as well as living expenses during the time that you will be studying for and taking the exam. 

Keep in mind that a US bar admission is not necessary for all career paths: if you are certain that you will return to your home country after the LLM, you may not need NY bar admission to advance in your career. Public international organizations, generally consider admission in a member jurisdiction an advantage, but do not necessarily require that it be a US jurisdiction. That said, if you are not admitted elsewhere, the NY Bar can serve as that admission, so it may be useful.

If you are unsure of whether NY bar admission will advance your career, consult with  Clara Solomon in the Office of Career Services (OCS) and Carolina van der Mensbrugghe in the Public Interest Law Center (PILC) to discuss your individual circumstances.

2. Review NY Bar Exam Eligibility Requirements

The NY Board of Law Examiners (NY BOLE) interprets and applies the Rules of the New York State Court of Appeals for the Admission of Attorneys and Counselors at Law (hereafter “the Rules”) on eligibility to take the NY bar exam.

Some students with law degrees from what NY BOLE considers to be purely common law jurisdictions will be found eligible to sit for the NY bar on the basis of their first law degree. However, many students will need both to complete the LLM degree and take particular classes during the LLM in order to establish NY bar eligibility.

According to Section 520.6 of the Rules, there are four basic requirements for establishing eligibility as a foreign-trained lawyer (See the  Foreign Legal Education section  of the NY BOLE website):

  • You must have fulfilled the educational requirements to be admitted to practice law in a country other than the United States by completing a program of law school study.
  • The foreign law school or schools that you attended must be accredited by the government or authorized accrediting body to award a first degree in law.
  • Your program of study for your foreign law degree must have been for an amount of time substantially equivalent to the period of study for a US JD at an ABA-approved law school and substantially comply with the instructional and academic calendar requirements set forth in Sections 520.3 (c)(1)(i) and (ii) and 520.3(d)(1) of the Rules.
  • Your study for your foreign law degree must be based “upon the principles of English Common Law” and the course of study must be substantially equivalent to legal education at a US law school.

Applicants may “cure” durational or substantive deficiencies (but not both) by obtaining an LLM degree at an ABA-approved law school in the US.  To qualify under the “cure” provision, applicants must:

  • Earn an LLM degree within 24 months of matriculation.
  • Take a minimum of 24 semester hours of credit in courses requiring classroom instruction.
  • A maximum of four credits may be earned in summer courses.
  • Coursework must be completed physically at the campus of the ABA-approved law school in the US. No credit is allowed for distance, correspondence or external study or for any DVD or online program or course.
  • Non-classroom credits, for example, those earned for Directed Research or for serving as a Research Assistant do not count toward the credits required.

3. Note Coursework Required During the LLM Under the “Cure Provision”

To establish NY bar eligibility under the “cure” provision, the LLM program must include a specified minimum number of credits for classes in four areas. For a detailed description of required coursework, refer to Section  520.6(b)(3)(d) .

If you need a US LLM to establish bar eligibility, the LLM degree program must include:

  • A: professional responsibility (2 credits);
  • B: legal research, writing and analysis (2 credits);
  • C: American legal studies (2 credits); and
  • D: subjects tested on the New York State bar examination (6 credits)

View the list of NYU Law classes that have been approved by the NY Court of Appeals to fulfill the above requirements.

The LLM degree program may—but does not need to—include credits in the following types of classes as a part of the overall 24 credits in classroom courses required:

  • Credits in clinical courses as long as the clinic course has a classroom component.
  • A maximum of six credits in other courses related to legal training (so long as the course is taught by a faculty member at the law school awarding the LLM or an affiliate school and the course is completed at a campus in the US).

4. Be Aware of NY Bar Admission Requirements

After you have passed the relevant exams, you apply for NY bar admission. As part of the admission process, you will need to show compliance with the following additional requirements both of which you may be able to complete before starting your LLM studies.

Skills Competency 

The Skills Competency requirement is applicable to students commencing the LLM in August 2018 or later; they must show competency to provide legal services in New York State ( §520.18 of the Rules).

To establish competency NYU Law LLM students must use the pathways described in subsections (a)(4) and (a)(5) of the Rule. These two pathways allow candidates to establish competence before or after the LLM through legal apprenticeship or legal practice in the US or another jurisdiction. For further information review the FREQUENTLY ASKED QUESTIONS FOR NEW YORK’S SKILLS COMPETENCY AND PROFESSIONAL VALUES BAR ADMISSION REQUIREMENT (PDF: 264 KB) .

Pro Bono Requirement

Under the New York State Pro Bono requirement, persons applying for admission to the New York State Bar must file an affidavit showing that they have performed fifty hours of qualifying pro bono service.  You will be required to complete the affidavit form (PDF: 247 KB) , including certification by your attorney supervisor, for the qualifying pro bono project(s) that you do. It is recommended that you complete the form(s) at the time you complete the pro bono work. You can find Rule 520.16 (PDF: 115 KB) , the affidavit form, and Frequently Asked Questions (PDF: 487 KB) on the NY Courts website.

If you do not fulfill the pro bono requirement in the year before you start the LLM, you may opt to do pro bono work during the LLM. In fact, pro bono work is a great way to connect to New York City’s rich legal community. Detailed information on the pro bono requirement , including how to search for pro bono opportunities, is available from NYU Law’s Public Interest Law Center.  If you are a current student or alum, contact Carolina van der Mensbrugghe, Associate Director, to discuss how to connect with pro bono opportunities.

5. Request Evaluation of Your Foreign Academic Credentials

All foreign-trained lawyers who wish to sit for the NY bar exam must show that their first degree in law meets certain requirements. To request evaluation of your foreign academic credentials: 1) complete NY BOLE’s online request form  (indicate that you are entering an LLM program at NYU School of Law); 2) request that supporting documentation be sent directly to NY BOLE.

Supporting Documents

Typically, after you complete the online form to request evaluation of your foreign law credentials, NY BOLE will send an email detailing the documents needed to support your request. 

The required documents, submission procedure, and additional documents that may be requested are listed on NY BOLE's webpages on Foreign Legal Education (See Roman Numerals VI – VIII).  Also see the instructions on the  website to Request Evaluation of Academic Credentials .

What to Submit

  • Official Transcript(s). Every law school you attended must send an official transcript to NY BOLE.  If you studied abroad during law school, also ask the institution where you studied abroad to send a transcript to BOLE.
  • Degree Certificate. If the official transcript does not clearly state the degree awarded and/or the date such degree was awarded, you must also furnish the degree certificate.
  • (a) If you are admitted to practice law in a foreign country, attach a copy of your admission certificate; or
  • (b) If you are not admitted to practice law in a foreign country, submit proof of the educational requirements for admission to practice law in your country and proof from the bar admission authorities that you have fulfilled these requirements.
  • Accreditation. Submit a written statement from the competent accrediting agency of your foreign government that the law school or schools you attended were recognized by them as qualified and approved throughout your period of study.

Important note: Documents from schools and bar licensing organizations should be sent directly by these institutions to NY BOLE.  Be sure that all documents submitted on your behalf include your NY BOLE identification number.

If you are not admitted to practice, you are asked to submit proof of the educational requirements for bar admission and proof you’ve fulfilled those requirements. You may submit a copy of the statute that governs bar admission, and request that the relevant licensing organization send a letter to BOLE including the statement that you have successfully completed the educational requirements for admission to practice law in the country where you received your foreign law degree. Some applicants request this letter from their foreign law school. It is up to NY BOLE (and not NYU Law) to determine what documentation is acceptable.  

Not all candidates need to submit documentation regarding law school accreditation. When completing the Request for Evaluation, you will find a drop-down menu including law schools which NY BOLE recognizes as accredited.  If the qualifying first law degree is from one of the schools listed, further evidence of accreditation likely will not be required.

Translation

If documents required by NY BOLE are in a language other than English, you’ll need to provide translations.  You cannot translate the documents yourself – they must be translated by an official translator.  You can send translations directly to NY BOLE; they do not have to be sent by the issuing school or other institution.

When to Submit 

We recommend that July bar-takers submit supporting documents to NY BOLE a year in advance of the July exam.

Your Law School and also perhaps the bar licensing organization in your home country will need to send documents directly to NY BOLE – these institutions make take some time to respond to your requests. Additionally, NY BOLE may find that a document you have submitted is insufficient and request further information. Respond promptly to requests for further information and ensure that the additional documentation requested is received by NY BOLE’s deadline of October 1 for the July bar exam and May 1 for the February exam.

In the event you don’t receive an email from NY BOLE confirming receipt of submitted documents, follow up with a phone call. After you supply the required documentation, you should receive an email message from NY BOLE stating that your file is complete and ready for review. If you do not receive that message, it is worth calling BOLE to find out what is missing.  Even after you receive that email, it is possible that BOLE will request further information, so again, we recommend sending documents as soon as possible.

6. Apply and Sit for the Uniform Bar Exam (UBE)

Most of our students who take the UBE choose to sit for the July administration of the exam after graduating from the LLM program. Some students choose, for a variety of reasons, to put off taking the exam until the following February.

Those taking the exam in July will usually begin studying for the UBE immediately after final exams are over and until the date of the exam.  Many of our JD and LLM graduates who are taking the NY bar exam enroll in a bar review course – these are provided by private companies with instruction delivered in a variety of modes.  Before investing in a particular bar preparation course, discuss the various options with others who have studied for and passed the UBE and consider your learning style and what will be most convenient for you.

Application

To apply to sit for the UBE, complete the online electronic application on the NY BOLE website. Please check the NY BOLE website for the deadline to file an application to take the July or February UBE.

The application fee for foreign-trained attorneys is $750. This fee is not refundable if a determination of your eligibility is not made in time for you to sit for the upcoming exam, or if you are found to be ineligible.

Taking the UBE

New York administers the UBE on the last Tuesday and Wednesday of both July and February in New York City, Albany and Buffalo. 

The UBE consists of three parts:

  • Multi-State Essay Exam (MEE) – Consists of six 30-minute questions. Areas of law that may be covered include: Business Associations, Civil Procedure, Conflict of Laws, Constitutional Law, Contracts, Criminal Law and Procedure, Evidence, Family Law, Real Property, Torts, Trusts and Estates, and Secured Transactions. The particular areas covered vary from exam to exam.
  • Multi-State Performance Test (MPT) – consists of two 90-minute skills questions covering legal analysis, fact analysis, problem solving, resolution of ethical dilemmas, organization and management of a lawyering task, and communication.
  • Multi-State Bar Exam (MBE) – a six-hour, 200 question multiple-choice examination covering contracts, torts, constitutional law, criminal law, evidence, and real property.

Testing Procedures

The New York Bare Exam Information Guide (PDF: 96.4 KB) gives a comprehensive description of testing processes and policies.

Also review the FAQ page for the UBE, NYLC, NYLE .

7. Complete the NYLC and NYLE

An applicant for admission in New York, must also take and complete an online course in New York-specific law, known as the New York Law Course (NYLC), and must take and pass an online examination, known as the New York Law Exam (NYLE).

The NYLC is an online, on-demand course on important and unique principles of New York law. The NYLC consists of approximately 17 hours of recorded lectures with embedded questions which must be answered correctly before an applicant may continue viewing the lecture. An applicant must complete all of the videos before the applicant may register for the NYLE. An applicant may complete the NYLC up to one year before, or within three years subsequent to, taking the UBE.

The NYLE is a 50 item, two-hour, open book, multiple choice test on New York laws and rules. The NYLE is administered online and offered four times per year. Please note you may not register for the NYLE until you have completed the NYLC. After an applicant has successfully completed the NYLC, the deadline to register for the NYLE is 30 days prior to the date of the NYLE.

The FAQ page for the UBE, NYLC, NYLE is also available on the NY BOLE website.

8. Register For and Take the Multi-State Professional Responsibility Exam (MPRE)

The MPRE is another required exam for admission to the NY bar. It measures the examinee's knowledge and understanding of established standards related to a lawyer's professional conduct. The MPRE is based on the law governing the conduct of lawyers, including the disciplinary rules of professional conduct currently articulated in the American Bar Association (ABA)  Model Rules of Professional Conduct (MRPC) and the ABA Model Code of Judicial Conduct (CJC), as well as controlling constitutional decisions and generally accepted principles established in leading federal and state cases and in procedural and evidentiary rules.

View  detailed registration information for the MPRE .  The test is typically offered in March, August and November.  Some LLMs opt to take the exam in August, after the July UBE.  If you plan to take the MPRE during your LLM studies, consider taking the exam in March, rather than November, so that you can use the spring break to prepare for it.

9. Arrange for NYU Law to Submit the Certificate of Attendance

For those who need a US LLM to establish NY bar eligibility, NY BOLE requires both a copy of the LLM transcript (showing completion of the requirements of an LLM degree), and a Certificate of Attendance certified by the Law School. 

To request that NYU Law send your transcript to NY BOLE, submit the NYU State Bar Request Form to the Law School’s Office of Records and Registration by May 31 (for July bar-takers).  You may send the form via email to [email protected] .

When you complete the application to sit for the NY bar exam, you will be directed to complete an online Certificate of Attendance Form. The Law School has access via an online portal to certify the Certificate of Attendance Form after you have graduated and earned the LLM degree. If you have submitted an NYU State Bar Request Form, the Law School will send to NY BOLE a copy of your transcript along with the Certificate of Attendance form.

If you are eligible for the NY bar exam on the basis of your first law degree alone, you do not need to complete this step.

10. Apply for Admission to the Bar and Complete Character and Fitness Review

In addition to passing the relevant exams and meeting the pro bono and skills competency requirements discussed above, you also must undergo an assessment of your Character and Fitness as a part of the application process to be admitted to the NY bar.  

Process to Apply for Admission

NY BOLE certifies you for admission to one of the four judicial departments of the Appellate Division in the State and sends you a “Notice of Certification.” The judicial department to which you are certified is based upon your address.

You will file an application for bar admission with the judicial department to which you’ve been certified. Processes vary across the four judicial departments. Depending on the department to which you are assigned, you may or may not be permitted to file an application for admission before learning whether you’ve passed the bar exam. (Regardless of whether you can file for admission before learning the results, you must pass the bar exam, NYLC and NYLE, and the MPRE in order to continue with the admission process.)

The application for admission includes: 

  • A questionnaire;
  • Affidavits of Good Moral Character; 
  • Legal Employment Affidavits from every law-related position you have held (including paid or unpaid internships, summer associate positions, volunteer work in law school clinics or elsewhere); 
  • Law school certificates for you to fill out and send to every law school you attended so that the law school can complete its portion of the form and send it directly to the judicial department; and
  • Affidavits showing that you have met the Skills Competency and Pro Bono requirements.

The questionnaire and other required forms are posted on the NY BOLE website.

Character and Fitness Interview

As a final step in the Character and Fitness Investigation, the judicial department will ask you to appear for an interview with a member of its Committee on Character and Fitness.  

Oath of Office

Applicants who are approved by the Court are asked to attend a swearing-in ceremony at which candidates appear before the Court on a specified date to take the constitutional oath of office as an Attorney and Counselor-at-Law.

Responsibilities to maintain licensure after becoming licensed to practice in New York are explained on the New York Court of Appeals website. 

© 2024 New York University School of Law. 40 Washington Sq. South, New York, NY 10012.   Tel. (212) 998-6100

IMAGES

  1. New York Bar Examination

    new york state bar exam practice questions

  2. New York Bar Exam Outlines

    new york state bar exam practice questions

  3. Bar Examination Questionnaire for Criminal Law.docx

    new york state bar exam practice questions

  4. 12 New York Bar Exam Subjects

    new york state bar exam practice questions

  5. New York Bar Exam Full Package

    new york state bar exam practice questions

  6. NY BAR EXAM ESSAYS

    new york state bar exam practice questions

VIDEO

  1. Virtual Tour: NYPL’s Equal Rights Amendment: A Century of Speaking Out

  2. Washington State: Bar Exam No Longer Required?

  3. ALM

  4. NYSBA Annual Meeting 2023 Recap

  5. What Black History Month Means to Our Members

  6. New York State Bar Association Task Force on Advancing Diversity

COMMENTS

  1. Past Exam Questions and Sample Answers

    office (518) 453-5990 fax (518) 452-5729 TTY: Call 711 (nyrelay.com) Phone Hours: Monday - Friday 8:30am - 4:30pm Eastern Time

  2. Bar Exam Practice Questions & Answers

    Practice Questions and Answers. Bar exam prep can be expensive, so to start you off on the right foot, we'll explore some free bar exam sample questions and answers, with a special emphasis on multiple-choice questions: 1. Torts Law: Understanding Liability and Damages. Question: Tom, in a moment of inattention, collides with Jerry's car ...

  3. New York Bar Exam

    The NYLE is a 50-question, multiple-choice, open-book, online exam. Each of the subjects covered in the NYLC will be tested on the NYLE. The NYLE may be completed up to one year prior to or within three years after taking the NY Bar Exam. Applicants must score at least 30 out of 50 questions (60%) to pass the exam.

  4. PDF New York State Board of Law Examiners

    owned by a New York resident. If the construction worker brings an action against the property owner in New York, what law governs the property owner's liability for failure to provide safe scaffolding? a) New York law, because the property owner is a resident of New York. b) New York law, because the action is pending here.

  5. PDF QUESTIONS

    1.b. Carlos is liable for any part of the unpaid balance of the purchase price if the partnership cannot pay for it. The issue here is whether a partner who did not authorize a contract is liable on the contract. All partners in a general partnership are jointly and severally liable for the partnership debts.

  6. PDF July 2021 Remote New York State Bar Examination MEE & MPT Questions

    Three weeks after the daughter's arrival in State A, the man sued in a State A court to establish his paternity, to gain sole custody of the daughter, and to obtain child support from the woman. The man had the woman served personally in State B. Under State A's long-arm statute, the State may exercise personal jurisdiction over a

  7. Bole- Official Page New York State Bar Examination

    The Board maintains a business office in Albany, New York with a full-time staff that oversees the bar examination. The bar examination is administered twice per year, on the last consecutive Tuesday and Wednesday each February and July. In 2023, the Board processed 15,804 applications for the bar examination and examined 13,440 applicants.

  8. Bar Exam Preparation Resources

    Mid-term Practice MBE Exam : Questions and Answers. MBE Final Review Course. New York Bar Notes : Lecture Materials. New York Bar Points : Subject Memorization and Review 2014. New York Subjects : Outline Materials. New York Workbook : Essays, Multistate Performance Test, Simulated Practice Exam. Multistate Bar Examination Civil Procedure ...

  9. New York Bar Exam

    Note: The New York State Board of Law Examiners additionally offers a Sample Questions document with 20 questions. How to Pass the NY Bar Exam. In order to give yourself the best chance of passing the New York UBE exam on your first attempt, it's important to take enough time to make sure that you are adequately prepared. New York Bar Exam Tips

  10. NY Bar Exam Review

    NY Bar Exam Review - CPLR. New York Trial Courts. Click the card to flip 👆. Supreme Court - general jurisdiction; cases should be heard in lowest court possible; matrimonial actions. County Court - criminal, up to $25,000 where all defendants reside or have business office. Surrogate's Court - decedent and inter vivos trusts.

  11. Law School Exam Practice Questions & Answers

    PLEASE NOTE: These series contain multiple choice questions designed for law school exams. They are not intended for bar exam study, although they may still prove useful. SERIES: Questions & Answers: Multiple Choice and Short Answer Questions and Answers. Questions & Answers: Civil Procedure. Questions & Answers: Constitutional Law

  12. New York Multistate Bar Exam

    Pass the New York bar exam with confidence! Start with our free New York Multistate Bar Exam practice questions below. We've selected 15 New York MBE free sample questions among the 7 categories of law tested on the New York Multistate Bar Examination: Civil Procedure, Constitutional Law, Contracts, Criminal Law & Procedure, Evidence, Real Property, Torts.

  13. Bar Exam Sample Questions for MBE

    Try Our Free MBE® Questions. Practicing realistic and current MBE sample questions is the best way to prepare for the bar exam, because if practice feels like the actual exam, then the real thing will feel like a practice. Take a sneak peak at these free sample questions to gauge the bar exam's difficulty. View MBE Sample Questions.

  14. Bar Exam Resources: New York

    New Book on Reserve. New York Bar Picture Book by Wela Quan. Call Number: KF303 .Q83 2016 (Reserve) ISBN: 1367197724. Publication Date: 2016-09-23. From the author's website: New York Bar Picture Book is an illustrated large format visual study outline for the New York State Bar Exam.

  15. AdaptiBar Online MBE Simulator & Prep

    Students choose only one correct answer - the best answer - from the four available choices. AdaptiBar's MBE Simulator and Prep gives you the confidence boost you need to pass the MBE portion of the New York Bar Examination. Our approach to licensed questions, in-depth explanations, and performance tracking ensures you are getting the most ...

  16. Sample Bar Exam Questions: 50 Free MBE Practice Questions

    The following practice test incorporates many of those seven subjects into the hypothetical problem-solving, multiple-choice questions. Most of the 50 bar examination practice questions are based on a hypothetical problem. That problem raises a legal conflict between disputing sides that must be resolved through the legal process.

  17. LL.M. NYS Bar Applicants

    All LL.M. candidates applying for the New York State Bar must complete the following steps: 1. Complete the 17-hour online course on New York law (NYLC) 2. Pass the 2-hour online examination on New York law (NYLE) 3. Pass the Multistate Professional Responsibility Examination (MPRE) 4. Complete 50 hours of pro bono service.

  18. Admission

    Each applicant for admission should visit the website of the appropriate Appellate Division of the Supreme Court by clicking on the link in the box below to learn of their individual requirements or forms. The telephone numbers and website links are as follows: First Department. (646) 386-5893. Second Department. (718) 923-6360. Third Department.

  19. Bar Exam Questions: Practice Bar Exam (MBE, MEE, MPT Review)

    This practice exam consists of 100 official questions released by the NCBE that appeared in previously administered MBEs. Does not contain Civil Procedure questions. The purpose of this exam is to familiarize you with the format and nature of MBE questions. 811-910.

  20. New York Bar Exam Eligibility and Admission to the Bar

    7. Complete the NYLC and NYLE. 8. Register For and Take the Multi-State Professional Responsibility Exam (MPRE) 9. Arrange for NYU Law to Submit the Certificate of Attendance. 10. Apply for Admission to the Bar and Complete Character and Fitness Review. §520.6 of the rules of the NY Court of Appeals for the Admission of Attorneys and ...